Howdy, Stranger!

It looks like you're new here. If you want to get involved, click one of these buttons!

Conditional Answer Choices for STRENGTHEN/WEAKEN

enzonabievenzonabiev Member
in General 44 karma
Hi thanks in advance if anyone can help. How should I deal with conditional answer choices in strengthen and weaken questions. Does it matter if the sufficient condition is triggered in the stimulus or does it not matter? Sometimes I have trouble with these answer choices because I'm hesitant to choose them since the stimulus doesn't indicate that the sufficient condition has been met even though the necessary condition will either weaken or strengthen the argument depending on the question stem.

Comments

  • DumbHollywoodActorDumbHollywoodActor Alum Inactive ⭐
    7468 karma
    It's hard to say. With LSAT, there's rarely hard and fast rules like that. Can you give us an actual question with pt number and section number?
  • enzonabievenzonabiev Member
    44 karma
    Thanks for the response, Ya ill give 2 examples where a conditional answer is correct and another where its incorrect. Do you happen to have the early lsat such as PT 4 and PT 6? I can post the question if you don't.
  • DumbHollywoodActorDumbHollywoodActor Alum Inactive ⭐
    7468 karma
    Actually you can’t post the question since that breaks copyright law and 7Sage’s TOS. I have all of the strengthen and weaken questions. Which section and question number?
  • enzonabievenzonabiev Member
    44 karma
    Oh sorry I didn't know that. The first one is from PT 4 section 1 #13. I was able to get it correct (D) by elimination but was hesitant due to not knowing if the sufficient condition had to be met. Does it not matter if we know that the researchers actually thought this way as long as it's a possibility then it weakens the argument? The next one is from PT 5 Section 3 #8 where i had trouble eliminating (E) even though (C) clearly weakens the argument. I guess the trouble I'm having is that I'm wasting time thinking about the conditional aspect of the answer and being unsure if it would actually affect the argument if its unclear that the sufficient condition has triggered. Is E incorrect because it doesn't matter that air pollution would increase if the total number of automobiles in use are increased as long as older automobiles are not the cause of the pollution? BTW Thanks a lot
  • J.Y. PingJ.Y. Ping Administrator Instructor
    14039 karma
    If you have access to the Question Bank, you can see them legally there: http://7sage.com/question-bank/
  • DumbHollywoodActorDumbHollywoodActor Alum Inactive ⭐
    7468 karma
    I see why you’re worried. A conditional statement is really just a hypothetical situation. A hypothetical could weaken an argument because it capitalizes on something the argument fails to consider. A hypothetical could strengthen an argument if it blocks a condition that would capitalize on something the argument failed to consider. We’re not really trying to trigger anything to make the argument valid in Sufficient Assumption questions or necessary with Must Be True questions.

    Ok, so for #13, like most weaken/strengthen questions, I want to break down the stimulus and find the flaw/gap. Finding the flaw first makes this such an easier process.

    Conclusion: Each academic discipline subscription decisions should be determined solely by a journal’s usefulness in that discipline, and that usefulness is measured by the number of times that journal is cited in other journals.

    Premise: Soaring prices of of journals have forced libraries to reduce their number of subscriptions.

    Flaw: It’s a weird stimulus because the gap is self-contained in the conclusion. What jumps out at me is the arbitrariness of equating usefulness with the frequency with which a journal is cited. Is that the only useful way to determine a journals usefulness? So I want to look for an answer that takes advantage of that flaw. In other words, we’re looking for an answer that explains how the number of times a journal is cited is not a useful way of deciding which subscription to keep.

    Elimination: (A) the nonacademic readership plays no part in this argument's flaw. (B)average length of journal article also plays no part in the argument’s flaw. (C) the genral public access to the journals plays no part in the argument’s flaw. (D) looks good. (E) these controversies are irrelevant unless they play a part in the citation process. But since the answer doesn’t say, we can eliminate.

    Selection: (D) This is the only one that addresses the gap that we pre-phrased. If that statement is true, then the number of citations is called into question as being useful. (D) is the correct answer.

    Probably more than what you wanted but I wanted to be thorough. I can get to the other question later, but I think I’ve answered your essential question: there’s no need to “trigger” the conditional somewhere in the stimulus for a conditional statement to Weaken/Strengthen or strengthen the argument.
  • enzonabievenzonabiev Member
    44 karma
    Thank you for such a thorough explanation. I just want to clarify a few things that you mentioned. So when an answer choice say "If this....then this..." I shouldn't worry about actually determining if the sufficient condition occurred, but rather focus on whether the possibility of it occurring affecting the strength of the argument? Should I focus on the necessary part of the condition in determining if it addresses the gap of the argument? Sorry for asking so many questions
  • enzonabievenzonabiev Member
    44 karma
    I guess I'm just a little confused on the comments regarding conditional answers weakening when it fails to consider an aspect. So for instance for #13 since the argument failed to consider that researchers may think that the journal in which the citation appears is not highly regarded by leading researchers then that would weaken the argument?
  • DumbHollywoodActorDumbHollywoodActor Alum Inactive ⭐
    edited October 2015 7468 karma
    Generally, you want to find an answer that the argument fails to consider that explains why the conclusion might not be so even though the premises are true. Specifically, answer D explains why deciding which journals to keep based on the number of citations it receives might not be very useful.

    As for "focusing on the necessary," I think you have to take the statement as a whole. Obviously in this example, the necessary condition is what does the most damage, but I'm sure there are other examples where the crux of the weakening portion is in the sufficient condition.

    I suppose if the sufficient condition were to contradict something in the stimulus, then you could eliminate that answer (because it would render that statement irrelevant..thank you Logic Games). But I don't think focusing on one side of a conditional statement over another is a great idea for these kind of questions.

    The key to doing well on Weaken and Strengthen questions is to stay flexible. Because they often deal with flaws that the argument fails to consider, the correct answer could seem to come out of left field.

    Hope this clarifies.
  • enzonabievenzonabiev Member
    44 karma
    Thanks a lot for the help!!
Sign In or Register to comment.